Hi ellyb,
Thanks for the question, and welcome to the forum!
You're asking a great question: how do we reconcile the fact that answer choice (B) strengthens the argument with the logical flaw that it seems to entail? Indeed, lack of evidence against a position does not prove the position to be true: for example, just because we cannot disprove the existence of God does not, in and of itself, prove that God exists. Note, however, that the Use of Evidence fallacy is triggered by the unwarranted assumption that lack of evidence against a position (or for a position)
proves that the position is true (or false). There is a critical difference between strengthening a position and proving it to be true, just like there is a difference between weakening an argument and proving it to be false. You appear to conflate the two, which is, ironically, a different logical flaw frequently tested on the LSAT.
Re:
Question 26 from LR2 of the December 2001 test, the case isn't that the author is arguing that answer choice (B)
proves the existence of a fifth universal force. That would indeed be an error in the use of evidence. However, if all of the previously established theories are compatible with the existence of such a force, this makes it more
likely that such a force actually exists. Our goal is to strengthen the conclusion, not necessarily prove it to be true: this is a Strengthen, not a Justify, question. By stating that all existing theories are totally cool with the existence of a fifth force, answer choice (B) eliminates a potential problem, namely, that the fifth force might contradict a previously established theory. This eliminates a potential weakness, thereby strengthening the conclusion of the argument.
Hope this clears things up!
Please let me know if you have any follow-up questions.
Thanks,